Sie sind auf Seite 1von 72

For more material and information, please visit Tai Lieu Du Hoc at www.tailieuduhoc.

org

GMAT Study Strategy

Revised 4/25/05

Authors: Daveformba, Ursula, GMAT Club, Stephen Bolton, Erin

Ive searched all over the internet and read all the major books on preparing for the GMAT. The main thing that current GMAT prep books do well is to help identify what youll face on the test. What they dont do well is to help you to know how to actually study each type of question and how to apply the strategies effectively. Ive struggled to try and figure out how take a more active posture in studying such that information is not only retained, but efficiency improved. Im tired of hearing, just do a lot of problems. Thats not useful. That was what people were telling me. One person, who attained a 740 on the GMAT at 46 years of age, the first time she took the test, stated how important it was to stay active and not be passive while studying. The point of this whole guide is on how to become an active learner for the GMAT and avoid being passive. (Read her Story here) As I was studying, I realized that I had to lay down some very specific strategies for myself to become more active in my learning process. This was because of a GMAT practice test I took where I got a 600. I had studied the Quant quite a bit, but not the verbal. For verbal, I went mostly off of what sounded right to me. I needed. I wanted to come up with a way to approach the rest of my studying more effectively because there seemed to be too much memorization or gut reliance going on in my test taking skills. I knew that just going by what sounded right wasnt the best way approach the real GMAT. This guide was created as a result of brutally asking myself specifics on how I could and would improve in each area of the GMAT. How does one move from, just do a lot of problems or study hard to studying effectively and most importantly. Getting results? As this is a guide that is being refined and added to, I hope youll join me in letting me know what you feel needs to be re-worked or further explained by you or me to improve it. This will be of great benefit to you and me as well as others that might need this in the near future. Over time, Ive received permission from several authors and teachers that have provided great strategy guidelines and have inserted them into this document. Im not sure how you have discovered this GMAT study guide as I have not advertised it anywhere. In any case, I say good for you because it shows that youre doing your homework. Hopefully, you werent just looking for the easy way out? Trust me, no matter how much you look, you wont find a study guide better than this on the internet. Its not that I searched myself and judged this guide to be the best. Rather, anyone whos creating anything thats even close is trying to sell you something. Im not. Theres no extended version of this guide that I sell. This is

For more material and information, please visit Tai Lieu Du Hoc at www.tailieuduhoc.org

just me helping others. I do only ask for two things as you use this guide. 1) I have included links to books and study guides where I do receive very small referral fees. Therefore, I ask that you use the links in this document to purchase the GMAT books youll be buying used or new to help support this web site you found this document on. (By the way, I have and always will only suggest resources that have been their weight in gold. I dont work for or receive kickbacks ever and never well. As you progress through this document and the associated links, I think youll see that Ive put in a significant amount of time into putting this all together) 2) Provide me with feedback on how to improve this document such as information you may want to submit to me. For example, I have received some feedback that they would like to see more specifics on Math prep. As this is a lot of work, Im taking my time on that, but have included some information in this latest revision. I have compiled a web page of various resources to file my research and organize it for easy browsing. You can find it at Future MBA Resources (FMR) . I have created one specific page just for GMAT resources. This will be where youll find a wealth of resources and free tests to help you on your journey. For example, I compiled a spreadsheet that lists the difficulty of every question in the GMAT Official Guide from GMAC the creator of the test.

Dave daveformba@gmail.com My Blog

Erin http://www.sentencecorrection.com/forums/index.php?act=Mail&CODE=00&MID=2

Daves Editorial Corner As Ive received quite a bit of e-mail asking me questions about the GMAT and managing difficulties in progress, Ive decided to add this new section that includes my responses.

GMAT Question Answered Dear Dave,

Thank you very much for putting your GMAT strategy guide on the net, it's the best so far! I am not a native speaker, I am struggling with my preparation. I feel like I am not progressing, can you please advise something on this regard? I really appreicated it.

For more material and information, please visit Tai Lieu Du Hoc at www.tailieuduhoc.org

Best Regards, "T"

Answer: Thats a difficult question to answer. Ill try my best. Im hoping that youre doing well in Math. If this is not the case.. then the answer may be in how you review and think through why you get the wrong answer and ESPECIALLY why you got the right answer. You have to do your best to commit to daily reviews of the ones you got wrong from the previous day. OG is the best book for practice questions. Review all the answers in it. Moreover, you should review all the ones youve been getting wrong at least once a week. If you keep getting them wrong.. then youre approach has been to just solve the problem.. rather than understand the concepts. If you're interested in which questions are difficult, medium or easy, you can check out the spreadsheet that I pulled from GMAC to help you. For English.. If you think about it verbal is a math skill too. There are rules and guidelines and many ways to approach the answer. You have to understand the basics. I started to do that in my study guide. You have to make sure you memorize the basics. Then it comes down to repetition and practice. Dont just try to do as many problems as you can when it comes to English. Understand not just why you got the question right.. buy exactly why the other choices were poor. Dont just say.. makes sense or sounds right. It's better to figure out which rule it broke or which concept the wrong answer didnt adhere too. By the way take a practice full exam not just verbal or math but a full exam at least 1 per week or every other week. Find any test you can get your hands on. It doesnt matter if the test is adaptive or not. And put yourself under the same time pressures of both the test taking time and the length of breaks. This part is crucial. What I described is the big picture approach for the GMAT. When it comes to discipline there are two things that I would suggest. 1) Take a class, but quit the class if the teacher sucks. They should blow your socks off otherwise, its a waste of your time. 2) Whether you take the class or not get to a place where you can focus and study with little or no distractions. I study at universities myself. For example, while I wrote my Business School application essays and studied for the GMAT, I went to Stanford at their 24 hour study center and took some drinks and sandwiches everyday. It was not unusual for me to get there by 6pm and

For more material and information, please visit Tai Lieu Du Hoc at www.tailieuduhoc.org

leave by 1am. Most Universities like Stanford dont require a parking permit after 5pm weekdays and no permit is required on the weekends. I called and e-mailed my friends that I wouldnt be around much because of my studies and I canceled a lot of appointments and activities in my life. I gave myself permission once a week to go see a movie after taking a practice test because my brain was fried anyway. I'm definitely not an expert at the GMAT, but in my humble opinion... I've written what I believe to be a good approach to studying for the GMAT. Not all ways work for everyone. I hope you find what works the best for you. But trying various methods such as what I've outlined will hopefully get you there.

Best wishes

The Hard Facts About Your GMAT Score See how the chart below shows GMAT scores in 3 Tiers? I think people generally have a vague notion that 3 tiers exists, but with no hard evidence like this chart... who's to say otherwise

For more material and information, please visit Tai Lieu Du Hoc at www.tailieuduhoc.org

Here's a follow-up to the last chart. Again 3 tiers of applicants regarding GMAT scores.

Here's an interesting graph from Kellogg's 2004 entering class. This is a good GMAT question possibility. If 5282 applied to the 2-year MBA class resulting in 469, what percentage of students were admitted with the various GMAT brackets of scores as shown in the graph? When you do the math (Taking into account that Kellogg admits 12% more than needed for instances where the applicant chooses another school), you get the following: 640 or less GMAT = 1056 apps with 42 accepted at 4% acceptance. 650-690 GMAT = 1584 apps with 158 accepted at 10% acceptance. 700-740 GMAT = 2059 apps with 252 accepted at 12.2% acceptance. 750-780 GMAT = 581 apps with 74 accepted at 12.7% acceptance. That means the 2004 class had 25% of all students who scored 700 or more accepted. However, the actual student population is actually 62% people who scored 700 or more. This is actually a great challenge and encouragement as I thought it was much worse. So 38% of the class scored 690 or less? That's cool. 700-740 is only 2% more likely to receive acceptance than those who had 650-690. Not much of a difference. I think this is why we ought not to kill ourselves for not getting 700 and for not killing our self if we do get 700 or more and don't get into a top Business School. That 2% can't hurt though.

For more material and information, please visit Tai Lieu Du Hoc at www.tailieuduhoc.org

Comments from a GMAT Instructor Last year, I received some feedback from a 12 year veteran Princeton Review GMAT instructor. Here are some comments he wrote in an e-mail to me that may be helpful for you. What we do for high scoring students to make them get their scores up (in a classroom setting) is to run classes that focus on ONE AREA. For instance, Statistics and GMAT Miscellany, SC, CR, Data Sufficiency. People at or above the 600 level have to revamp their study plan. Random info dump won't be effective anymore. If you can get a 600, then you basically know most of the information there is on the test. What you are probably lacking is a systematic way of using that information and an understanding of good GMAT strategy.

I told the instructor that I wanted to focus on probability a bit more. His reply on that was as follows: You are basing some of your plans and concerns on forums. In the last 5 "Test Recons" there have been an average of 1.3 probability questions per test. 1.3 ... think of all the energy and time dedicated to probability in the forums and think of what kind of wasted effort is poured into that subject. Forums are 95% noise, 4% good intention, and 1% information. If you think of it that way you will actually look at them with an entirely new eye and perhaps glean the little there is there to be had. Test prep companies sometimes fall for the same sort of paranoia. They respond to the concerns of their students and when those students' fears are being fed and influenced by the forums a nasty little spiral starts to form. Ask how many people teaching the GMAT took the GMAT in the last 6 months I think you will be shocked and dismayed. While I do not think that someone has to take the test often to be a good teacher, a good teacher will take the test often to be able to screen the noise. The graphic below is part of how I assess student needs. Each red zone is a plateau. It is a score range that is REALLY hard to break through. You are in the plateau that the course is designed to get people to. Don't believe the nonsense out there. There is no super effective one-size-fits-all solution to maximum GMAT improvement. Achieving and moving through each plateau has its

own unique set of needs and concerns.

For more material and information, please visit Tai Lieu Du Hoc at www.tailieuduhoc.org

What youll find in this document 1) GMAT online forum resources and why they are helpful. 2) Study habits of 19 people who achieved 700+ GMAT scores. 3) How to track your progress and improve your review process 4) What kind of schedule should I use? What order should I go through the material? 5) Ongoing help resources Daily e-mail questions 6) What GMAT resources should I use? In other words, what books and resources out there are good and what sucks. 7) Should I use a Test Prep Company? 8) What kind of study schedule should I use? 9) A summary study guide for each section of the GMAT 10) A study guide on probability. There just arent any books out there that cover probability well.

GMAT online forum resources and why they are helpful A couple of websites I found in my research were www.testmagic.com and www.gmatclub.com. Both are tremendous resources because you can post questions there and people will help you to understand why one answer is right and another is wrong. You should see the generous amount of time people take to post responses. One thing I noticed was how a few people who did well on the actual GMAT mentioned how the forum benefited them so much. They went on to mention how they stayed active in the forums. A good forum is created when people take the time to explain an answer and not to just simply try to guess at the answer or write out a quick formula to show how you got the answer. How did you get the formula? What led you to start out with certain steps? What concepts were used in your solution? Putting your answer in this kind of context is helpful and most importantly forces the person who is posting the answer to explain the answer through fact and theory. To write out the answer requires a reasonable understanding of the concept being tested. To explain the answer to someone is literally like being a teacher. Teaching a topic is the best method of learning. Now thats active learning! Heres an example of a typical posting.

For more material and information, please visit Tai Lieu Du Hoc at www.tailieuduhoc.org

Posting Hi Guys! Help me with this one. Q. What is the volume of a certain rectangular solid? (1) Two adjacent faces of the solids have areas 15 and 24 respectively. (2) Each of two opposite faces of the solid has area 40.

I think statement (1) alone is sufficient to answer the question. If not, please explain me why? Thanks XXXXX

Reply #1 Statement (1) alone is not sufficient as we can get multiple results for volume: Face 1 can have an area of 15 with sides 15 & 1: 15 * 1 and Face 2 can have an area of 24 with sides 1 * 24. The volume would be 15*1*24 = 360

Face 1 : 5 * 3 Face 2 : 3 * 8 Volume : 5*3*8 = 120

Statement (2) alone is not sufficient as we have info only about 2 sides. Combining Statement (1) & (2), Areas with 15, 24 & 40 correspond to three sides with lengths measuring 5,3 & 8. Ans should be C. Please let me know the correct answer.

Reply by Original Poster The answer was infact C.

Reply #2 According to B each opposite face of solid has an area of 40.So all the faces of the solid are equal in area.So the solid is a cube.So each edge is (40)^1/2 long .So we can definitely find volume with B.So "B" should be the correct answer. Please let me know if I am missing some point over here. Reply #3

For more material and information, please visit Tai Lieu Du Hoc at www.tailieuduhoc.org

Statement 2 just says that each of two opposite faces of the solid have an area of 40. As therez no info about the remaining 4 faces of the cube, this statement alone is insufficient. Thanks, XXX

I found about 19 people who scored 700+ and found quite a few similarities in their study habits and what books they chose to prepare with. Here is a summary of that analysis.

Books used (98% used the following) 1. Kaplan GMAT book 2. Princeton Review book 3. Official Guide for GMAT 4. Power Prep Tests 5. Manhattan GMAT Study Guide Books (New!)

Book/Resources used (About 4-5 people used the following) 1. Kaplan 800 - Hard GMAT questions - more than a few argued that this had the hardest questions they found. 2. 800-score tests A good resource for tests. 3. GMAT Plus A good resource for tests. 4. DeltaCourse - primarily a combination, probability, permutation advanced study guide with plenty of sample questions to practice with. 5. Barron's - All said study guide part sucked, but questions were useful. 6. Verbal Workout for GMAT -Princeton

Books/Resources all said sucked. 1. Arco Master the GMAT CAT Many of the answer are wrong. But the study guides are good. 2. Petersons 3. Crack-GMAT Dont waste your time or money on this one according to many. 4. Kaplans in-person GMAT prep course a few who did it said that it didnt help.

For more material and information, please visit Tai Lieu Du Hoc at www.tailieuduhoc.org

Study tips that every single 700+ scorer mentioned. 1. Record all mistakes on wrong questions and revisit them no sooner than 5 days. If still getting wrong, then zero on the subset's and go for more help. 2. Always read the explanation in the book to see why you get it wrong and take enough time to really understand the concept. 3. Pick a date and register. Don't study then register. There was a tendency to not take prep time as seriously. 4. Quite a few went through OG questions more than once. Not all questions on the repeat though; Mainly verbal sections and Quant that were areas of weakness. 5. Wrote practice essays at least 20 times under timed conditions. 6. Visit the test center before the day of test. Many did so a week or two before. 7. Time spent by most averaged between 3 and 4 months. 8. All averaged 2-3 hrs per day and no less than 10-12 hrs on the weekends. 9. Took advantage of every break during the GMAT. Take a bathroom break, some washed their face to wake up, got fresh air. 10. Study in blocks. One person mentioned studying in 80 minute blocks without getting up to build stamina. Another person mentioned studying in 75 minute blocks with exact numbers of questions that are on the real GMAT to build stamina and skill. Many agree that the ability to handle the time constraints is nearly as important is solving the problem. Scores in the actual GMAT are worse if you leave an answer blank than if you get the answer wrong.

Interesting note: The Official Guide for GMAT contains questions that are in the 550 to 650 difficulty range.

Lets Get Started The first step is to get a bit analytical. When going through questions, use either one of the following tracking sheets. http://home.comcast.net/~dave.kim/GMATTracker.xls http://home.comcast.net/~dave.kim/GMATAnswersForm.zip

What this will help you do is track your progress in a way that can tell you in a measurable way whether carelessness or lack of skill caused a wrong answer. In other words, using this sheet will help you to zero in on the what you can target for improvement. Instead of saying, I need to improve in math youll be able to say, I need more work in probability, more work in stamina or

For more material and information, please visit Tai Lieu Du Hoc at www.tailieuduhoc.org

more work in seeing various forms of misplaced modifiers in sentence correction questions. One thing is for sure, most people find that 50% or more of the reason why they get a question wrong is due to carelessness, or a loss/lack of stamina. Without a good tracking system, you may not be able to measurably know by how much this is true or whether its something else. Its also good to use the notes area frequently to note the kind of subcategories of question types or if it was a concept you didnt know or a concept you did know, but werent solid on yet. When you start out, its helpful to spend a week or two un-timed. Then as you progress, start putting yourself under moderate time pressure to at least record how long its taking you to do a particular question. It would be completely ridiculous to record how long it took you to answer every question. Its far better to track yourself in blocks of time. For example, with 12 questions, you should have X minutes left and with 24 questions, you should have X minutes left and so on. Another excellent tracking method is to put one, two or three Xs in the slow box if you took longer than real GMAT time conditions to answer the question. This is one strategy I rely on heavily. I use both block time tracking and slow progress tracking actually, but find the slow progress tracking the most helpful.

Another good strategy is to go through questions in the following manner. 1) Do about 40 questions at a time. In this way, you should be giving yourself 80 or so minutes to complete them. This will help you to get used to doing questions in blocks similar to the GMAT. This will build your physical stamina and build longer periods of mental toughness and focus. 2) Check your answers and DONT look at the explanation of the answer. Circle the right answer in red and the sheet. 3) Go back, and use the teaching method as explained earlier to resolve the problem. Note whether you find the reason why you got the answer wrong as careless or due to concept error in the chart. A concept error is defined as an error where you didnt understand the theory enough to know which steps and in what order to solve the problem. It could also be a relevant piece of knowledge that you forgot or havent learned yet. See the sample posting above to get an idea of what a teaching method solution looks like. 4) Next go through the explanations to try and understand if you your methods were right or if the book offers an alternative solution method. There is more than one way to explain or solve many problems, so dont feel like you have to follow the particular explanation in the book. If your method solved it, and you can repeat it, then stay with what you know as long as it holds up to scrutiny when slight changes in the problem dont allow your method to work. In which case, youll need to adapt the book concept or dig deeper into

For more material and information, please visit Tai Lieu Du Hoc at www.tailieuduhoc.org

asking yourself whether you really understood the concept to begin with. By the way. Its really helpful to ask yourself two questions when youre done with each test problem resolve 1) would I be willing to bet $20 that I really understood the concept being tested? 2) Did I write out my explanation well enough that most anyone could look at it and understand how I solved the problem?

Ongoing Help You can have e-mails of GMAT questions sent to you on a daily basis. The two popular ones are Delta Course and Manhattan GMAT For Delta Course, go to http://www.deltacourse.com/gmat/gmat-email.asp For Manhattan GMAT, go to http://www.manhattangmat.com/ Select the Student Center menu and create a profile. At the bottom of the profile registration, youll be asked if you want the daily question or not. This is free by the way. What Resources should I use? Kaplan Workshop This is the CD you get when you sign up for the KAPLAN class. Its awesome. Both the Quantitative and Verbal reviews are helpful. I struggled with Sentence Correction and the Workshop exercises and walkthrough started from the basics and had mini-quizs to drill you on the material as you went through. The number properties review was very helpful as well. You can get it at scoretop http://www.scoretop.com/bbs/uploads/workshops.rar. Its a compressed file. If you dont wave winrar to decompress it, you can download it at www.rarlab.com for free. The Official Guide to the GMAT Can be obtained from Amazon Manhattan GMAT Prep Books This is by far the best set of study guide books for the GMAT on the market. Where Kaplan, Princeton Review, and the Official Guide all fall short is in the study prep material. I browsed a set of the books from a friend. I liked them so much that I went ahead and bought them. Princeton Review, Cracking the GMAT 2005 Can be obtained from Amazon Kaplan GMAT 2005 Can be obtained from Amazon DeltaCourse Only available online Powerprep Tests ETS, the company that makes the GMAT. GMAT Tutor has a good Math basics reference sheet to help you memorize formulas and math basics To see if your GMAT essay abilities are up to snuff, you can use GMACs services to review some sample essays. Theyll use the exact same software that is used on the GMAT to review and score your essay. You can get it at:

For more material and information, please visit Tai Lieu Du Hoc at www.tailieuduhoc.org

Test Magic www.testmagic.com - Excellent forum to get questions answered or explained. GMAT Club www.gmatclub.com Similar to Test Magic GMAT Club Prep material - Its pretty good. http://daveformba.blogspot.com my weblog. Obviously, this is where you got this guide from, but youll find that I make enough updates and additions to make it worth your while to check it regularly.

Scoretop.com If you dont have access to GMAT software or your CD got fried, go to http://www.scoretop.com/forum/forum_topics.asp?FID=6 Future MBA Resources This is my GMAT and MBA resources web site.

Thats a lot of resources isnt it? Heres a short summary of how to approach the Resources Study Guides. o o Manhattan GMAT prep books ($150) and worth it Kaplan or Princeton Review. Both do a reasonable job on the Verbal section of the GMAT. However, both are weak in Sentence Correction. o This Study guide you have before you. (Youll find that SC (Sentence Correction)) is highly emphasized. o o Kaplan Workshop DeltaCourse Without a doubt, the best resource for probability, combination type problems. Practice Problems o o o o o Practice Test simulations o o o PowerPrep 2 free simulations Score800 excellent resource of test like situation Arco CAT- Manhattan GMAT sells a version of this on their web site that is put together VERY well. o GMATClub Challenge Hard as heck math problems on a timed schedule. And its free. Personal tutoring The Official Guide This is a must buy. Kaplan and Princeton Review books. Check my blog for practice problems resources. Deltacourse (for math)

For more material and information, please visit Tai Lieu Du Hoc at www.tailieuduhoc.org

o o

See my Blog for lists of GMAT companies that provide private tutoring. You can get free tutoring from your peers through GMATClub.com, Testmagic.com and Scoretop.com. Theyll answer your questions and show you 5 different ways to answer the same problem. And the 3 forums are free.

Should I use a Test Prep Company? Its really up to you. If you find that you struggle with discipline or life is really busy. Then you should. Highly recommended is Princeton Review (www.princetonreview.com) and Veritas (www.veritasprep.com) If you have the money and desire more personal instruction, check out Manhattan GMAT (www.manhattangmat.com). Theyre very good.

What kind of schedule should I use? What order should I go through the material? Again, its really up to you, but here is a basic plan that most people have used that showed great results. 1) Go through the Kaplan Workshop FIRST. Its the best Basic review that Ive found of Math and Verbal. 2) Go through the Princeton Review Quant and Verbal Study Guide section 3) Do at least 50 questions of each type. 4) Take the one Princeton Review Test from the CD. Use it as a reference point to see where your at. Do it under the strictest time conditions. 5) Finish the Princeton book 6) Get the Deltacourse material and go through it. 7) At the same time, begin going through the Kaplan questions. 8) At a MINIMUM take one full GMAT practice test a week. Whether its Kaplan or Princeton. Save the last Powerprep for when your about 2-3 weeks away from the test. The Powerprep tests are well known to provide results quite similar to the real GMAT. The Kaplan ones are traditionally harder and therefore give lower average scores. Dont worry about the scores if they are lower. If you are worried, go to http://www.gmatclub.com/content/resources/estimator/index.php. There, youll find a score estimator. It will take the Kaplan score you get and average it out to a more realistic score. 9) Go through the Official GMAT Guide. Do all the questions.

For more material and information, please visit Tai Lieu Du Hoc at www.tailieuduhoc.org

10) If you have the time, use the GMAT Plus material for an additional source of questions to go through. 11) Take time to do a weekly review of the verbal and quant formulas and reference sheets you made. www.gmattutor.com has the beginnings of an excellent Math Basics reference sheet. Bar none, the Princeton Review for Verbal is your best bet. See below for my summarized version of it. 12) Do at least a dozen practice essays. Do each one under timed conditions. 13) Dont study the day before the test. And make sure to visit the test center at least a week before the test. Bring ear plugs to the test center if noise will be an issue. Get used to studying with them if you do though. 14) Create regular review sessions where you go over all the questions you got wrong on a weekly basis. Make a binder or something similar.

Problem Solving and Data Sufficiency Study Strategy Kaplan and Princeton are good for most topics except for Combination, Permutations, Probability and Counting. They dont cover these Quantitative math skills very well. The best resource Ive found is from Deltacourse Their material is highly focused on these topics. It was extremely helpful. For $27, I wouldnt even hesitate about ordering it online. You cant buy it in bookstores. For a scaled down version of this, go to Appendix A of this Guide. For every Data Sufficiency and Problem Solving question you get wrong, write out in long hand the solution. Dont just write out the formula. This will help you process the theory being tested. Explain why you did certain steps. I call this the teaching method explanation.

Heres a number Property table from Kaplan Workshop that neither the Kaplan or Princeton books provided and was extremely helpful as I realized that there were number properties I didnt remember or recall having learned years ago.

Number Properties Integer 0 1 2 7 -3 .5 X X X X X X X X X X X X X Positive Negative Even X X X X Odd Prime

For more material and information, please visit Tai Lieu Du Hoc at www.tailieuduhoc.org

Pi 6 -19/2 51/3 Even negative x Odd negative Odd negative + Odd negative Even Positive Odd Negative Odd negative / Even positive X X X X

X X X X X X X X X X X X X

Points of emphasis: O is an integer, it is neither pos of neg. Negative numbers are not prime 0 and 1 are not prime

Reading Comprehension Study Strategy 1) Aggressively read each paragraph for its main idea. If you cant write down in a few words what the point of each paragraph is, you werent reading actively enough. You should jot down the following. a. Main idea or primary purpose b. Organization/Structure c. Tone or attitude of author (if applicable)

Note: Be careful to not write facts down. Itll bog you down and usually results in a loss of the big picture and moves you to focus to much into the details.

2) Note any trigger words, same train of thought words, yin-yang parallelism. See the Princeton Review book for more explanation. 3) Weed out possible disputable answers. Vague, wimpy answers are often correct over stronger statements. ETS doesnt want to get many complaints that a particular answer that was strongly stated, that exceptions could arise. ETS would rather play it safe. 4) Minority passages are often positive in tone and answers tend to be positive in tone as well. Again, ETS, wouldnt want to look prejudiced. 5) Always eliminate bad choices first before answering. Youll almost always be able to narrow down to 2-3 and that significantly improves your odds of getting the question right.

For more material and information, please visit Tai Lieu Du Hoc at www.tailieuduhoc.org

6) Read the entire passage before answering the questions. Other books say skim, but its not always successful with more difficult passages where minute details change meanings of the passage and could get you going down the wrong path on inference or main point questions. Give yourself 3 minutes or less. 7) For Inference questions, (Note: these are usually the hardest of all RC questions) go find the general area being referenced. Read a bit above or below it and then make your choices. Dont go by memory. This is going to cause more problems that be helpful with saving time in the long run. Your answer should never contradict the main point of the passage. 8) Most people get main point and inference questions wrong so focus more carefully on these. 9) "According to the passage/author" question type of questions. Whenever you see this question, tell yourself, "Stop and stop thinking. I need to FIND, not think." 10) For main point or central idea type of questions, re-read the first and last sentences of each paragraph before making elimination choices and answering. Getting the overall structure is really helpful before answering. 11) In Summary: Consider weeding out answers that a. Are disrespectful to others/professionals. ETS doesnt like to be disrespectful. b. Too strong an answer. Use of words like only, definitely, positively c. Condone/approve prejudicial attitudes. ETS doesnt like to be disrespectful.

12) For a much more detailed strategy for Reading Comprehension, go to Appendix D

Sentence Correction Study Strategy Overall strategy 1) Memorize the 8 Princeton Review methods recognize bad sentences. Ive summarized the key points below. The Kaplan Workshop, Princeton Review/Kaplan and Arcos Sentence Correction study sections are the key resources that Ive learned the most from. And believe me I had a lot to learn. 2) Initially, do 20-30 questions and explain what error you saw in the sentence or didnt see. For wrong answer choices, list/explain specifically what Princeton Review rules this answer choice violated. Sometimes, its a combination. Also explain why the sentence was tricky if its a difficult one. For example try to identify if there were long prepositional phrases between a verb and subject agreement or a long adjective modifying phrase instead of an easy single word modifier.

For more material and information, please visit Tai Lieu Du Hoc at www.tailieuduhoc.org

3) For all future questions, make sure you write, verbally, or mentally talk through the rule that was violated. Make notes on a scratch piece of paper. If you pick an answer and cant explain EXACTLY what method the answer corrects, then you are just guessing or going off of what sounds good--- Therefore, reinforcing a bad habit.

Spotting bad sentences is the key to doing well on sentence structure test questions.
1) Pronoun error
a. Plural and single once you start with one, you need to stay in the same quantity.

Singular Pronouns (Memorize these. Period) Hint: Do you see the categories I setup? Its SANE to memorize this Some--Any No Every-Everyone Everybody Everything Anyone Anybody Anything Someone Somebody Something No one Nothing Nobody Whoever Whomever His Either Neither One Each

Be aware that group, jury, team, country, family are singular. Society today uses them sometimes as plural. This is because these act as a single unit when they do something.

Plural Pronouns (Memorize these. Period) Both Few Their Others Many Several

To memorize this, try Frightening TOMBS

For more material and information, please visit Tai Lieu Du Hoc at www.tailieuduhoc.org

Singular and Plural Pronouns depends on whether the noun is singular or plural (Memorize these. Period) Some More Most All

The plural and singular clause error When two nouns are in the sentence doing an action together but they are linked with Along with Together with With As well as In addition to Accompanied by this does not make the following action they do plural. Only and can take the two singulars and make their action plural. For example Janie, with her poodle limping behind her, walks to the dog park. Explanation: Janie is singular. The poodle is singular. They both do the action together, but the use of with means that we need to keep the verb singular. Walks is singular and Walk is plural Remember, a verb that ends with an s is singular. b. Pronoun reference error- referring pronoun is not correctly placed. c. Relative pronouns are often used incorrectly today. 1) Referring to things or animals that, which 2) Referring to peoplewho, whom 3) They be careful that you dont use this unless youre positive there is a referring noun. Today we often use they to replace the use of a proper noun which it is not. Its a Pronoun.

2) Misplaced Modifier (modifiers must stay close to home)


a. Definition: Sentences that begin with a verb, adjective+verb, adjective phrases need to be followed by the noun or pronoun they are modifying. Usually end with ing. b. Example: Coming out of the department store, Johns wallet was stolen.

For more material and information, please visit Tai Lieu Du Hoc at www.tailieuduhoc.org

Coming is the modifier. Was johns wallet coming out of the store? Incorrect sentence. Possible solution to look for: a) Correct the reference b) Put a noun or pronoun into the 1 part of the sentence turning the 1 part into an adverbial clause. c) Correct answer might be, As he was coming out of the department store, Johns wallet was stolen. Alternate could be, As John was coming out of the department store, his wallet was stolen. Absolute Phrases: Introduction 1. Introduction Definition and rules. An absolute phrase is a modifier (quite often a participle), or a modifier and a few other words, that attaches to a sentence or a noun, with no conjunction. An absolute phrase cannot contain a finite verb. Absolute phrases usually consist of a noun and a modifier that modifies this noun, NOT another noun in the sentence. Absolute phrases are optional in sentences, i.e., they can be removed without damaging the grammatical integrity of the sentence. Since absolute phrases are optional in the sentence, they are often set off from the sentence with commas or, less often, with dashes. We normally explain absolute phrases by saying that they modify entire sentences, rather than one word. This is an important concept, since many similar phrases that we work with modify other words. For example, adjectives modify nouns, and adverbs can modify verbs, adjectives, and other adverbs. That said, however, in some cases, it seems to make more sense to say that absolute phrases modify nouns. We will look at some of these examples a bit later.
st st

First, let's look at some examples of absolute phrases: Examples of Absolute Phrases: The absolute phrase look like this: Her determination stronger than ever, Nexisa resolved not to give up until she had achieved her dreams. The sun shining bright and the pale blue sky forming a backdrop of the Sacre Coeur, Carl stepped into his future as a traveler and observer. We finished the hearty meal quickly, our appetites satisfied, our minds at peace.

For more material and information, please visit Tai Lieu Du Hoc at www.tailieuduhoc.org

All things being equal, the active voice tends to be correct more often than the passive on standardized tests.

Please notice that in every case the absolute phrase provides some sort of information that works to put the whole sentence or idea in context. Please also notice that the absolute phrases themselves do NOT contain verbs, nor are they connected to the main sentence with a conjunction. Finally, please notice that the primary components of most (but not all) of these absolute phrases are a noun + a modifier, although it is possible to use only a modifier. Here is the next pattern we should know:

2. noun + participle This is one of the most common ways to form an absolute phrase. It might be helpful for some people to imagine this pattern with a verb between the noun and the participle. For example, if you say The question was still unanswered, you have a complete sentence; if, on the other hand, you say The question unanswered and you then attach that phrase to a main sentence, then you have an absolute phrase. Here are some examples. The absolute phrases look like this. The question still unanswered, the teacher decided to address the confusion of her students more closely. The train running late, we decided to get off at the next stop and take a taxi home. There are many industries in California vital to its economy, with technology being one of the most important. Compare these sentences with the verbs and conjunctions in them: The question was still unanswered, and the teacher decided to address the confusion of her students more closely. The train was running late, so we decided to get off at the next stop and take a taxi home. There are many industries in California vital to its economy, and technology is one of the most important. Similarly, having + past participle is often so semantically similar to the sentence without it that many sentences are written without having + past participle. An example would be very good here:

For more material and information, please visit Tai Lieu Du Hoc at www.tailieuduhoc.org

Having been chosen to head the committee, Angus Ng thought about how he could help raise money for his chess club at Harvard.

This sentence could look like this: Chosen to head the committee, Angus Ng thought about how he could help raise money for his chess club at Harvard. This concept is important for the Sentence Correction section of the GMAT, so if you're preparing for that test, pay attention to this!

3. noun + adjective Another pattern is to use an adjective after the noun it modifies. Look at these examples: Their meal still not ready after 45 minutes, the hungry and angry customers left the restaurant. His hat in hand and pride in check, Horace asked his former boss for his job back. The previews still showing, Kelly and Chris decided to leave the theater and enjoy the sunny day.

3) Parallel Construction a. Is there a list? b. Is the sentence in two parts? Both types must have parallel types of verbiage. to ____, to ____ or ate _____, slept ____, drank ____. Bad construction might look like to ____, _____ or ate _____, sleep _____, drank ____. c. Comparisons must be logical Find the two things being compared and see if the sentence is structured in balance. Dont be afraid to consider changing verbs or adjectives to get the balance. d. Parallelism is not just about clauses, but verb usage Example: -ing and ing, to to , either .. or, neither . nor. e. The words "like," "unlike," "similar to," "asso", when and "in contrast to" are the most common indicators of comparisons. In comparisons, compatibility is determined by subject matter. For example:

For more material and information, please visit Tai Lieu Du Hoc at www.tailieuduhoc.org

As domesticated animals, indoor cats typically lose their ability to hunt for their own food, so too do domesticated dogs come to rely exclusively on their owners for sustenance. Here, domesticated cats are compared to domesticated dogs, and the comparison works because they are both domesticated animals they are like terms. Whenever you see a comparison being set up in a sentence, check to see that the terms of the comparison are compatible. 4) Verb Tense a. Is the whole sentence in the same tense b. Some major categories of tense: Dont need to memorize types. Just be familiar Present tense Simple Past a day. Present Perfect Past Perfect Future example: He has walked. example: He had walked. example: He will work. example: He walks three miles a day. example: When he was younger, he walked three miles

Present Perfect Describes action that began in the past but continues until the present Key identifier has have Sidenote: Sometimes used when deadline exists. Past Perfect Describes action that started and stopped in the past Key identifier had Present Progressive Used as emphasis by the speaker that the action is happening this very minute Key identifier verb to be + a verb with an ing ending. Perfect Progressive Occupies more than one moment in the past. In other words, ongoing for a period of time. Key identifier had been c. Two events that have taken place , are taking place or will take place at the same time must have the same tense in the sentence. d. Passive verbs begin with the form of to be (Example: to be, were, was) and end with a different verb in the past tense.

5) Subject-Verb agreement errors.

For more material and information, please visit Tai Lieu Du Hoc at www.tailieuduhoc.org

a. Singular + Plural agreement. Do the two agree in plurality? Can be made very complex when prepositional phrases separate verb from noun/subject by 5 or more words. Easy to overlook cross referenced subject-verb relationship. b. To tell if a verb paradigm is plural or singular. 1) Mentally put They in front of the verb 2) Mentally put He in front of the verb plural singular

c. Verb plurality: (This comes up A LOT on the GMAT) Adding an s to the end of an adjective makes it singular. For example, dislike Subject/Verb Inversion So you already know that the GMAT test is an adaptive test, meaning that your score goes up or down depending on the difficulty of the questions that you answer correctly or incorrectly. On the sentence correction section of the GMAT, the questions that test you on subject/verb inversion tend to be the harder questions, and are therefore worth more points. So, to raise your GMAT score, you should be very familiar with most or all of the items on this list! plural dislikes singular

There are at least eighteen types of inversion: Type 1. neg intro Examples Never do I sleep. Only at night can I study. In no way could I help you with your Japanese grammar question. I believe that only rarely will I need your help. Not until I got home did I realize that my shoes were untied. Notes Question form is obligatory. Used with all verbs. This one is very common on the TOEFL and somewhat common on the GMAT and GRE. We need to learn the various types of words and phrases that require this type of inversion. Notice that sometimes the inversion occurs right after the neg intro form and sometimes it occurs in the next subject and verb.

For more material and information, please visit Tai Lieu Du Hoc at www.tailieuduhoc.org

Type 2. intro adverbial

Examples Into the room ran the lady. First comes love, then comes marriage. After A comes B, then comes C, next comes D. Down came the rain and washed the spider out.

Notes Inversion is optional. Used with be-verbs, linking verbs, and verbs of direction. This one is less common on the TOEFL, but more common on the GMAT and GRE. Notice that sometimes we have an adverb, like first and down and sometimes we have an adverb phrase like into the room or after A. These adverbs and adverb phrases usually show location or direction. This type of inversion usually only occurs with be-verbs, linking verbs and verbs that show direction or movement, like come, go, run, etc.

3. intro ed

Found in San Francisco is Lombard Street, the so-called crookedest street in the world. Lost among the old tables and chairs was the priceless Victorian desk. Located between San Francisco and Marin County is the Golden Gate Bridge.

Inversion is obligatory. Used with be-verbs. This one is very common on the TOEFL, GMAT, and GRE. This type of inversion usually occurs with be-verbs, but sometimes with linking verbs. Notice that the phrase is the complement of the be-verb. Inversion is optional. Used with all verbs. This form of inversion is common on the TOEFL, GMAT, and GRE. We normally only have inversion here if we are comparing subjects

4. comparatives

Cheetahs run faster than do antelopes. You speak Chinese better than do I. Jessica is more interested in Computer Science than is

For more material and information, please visit Tai Lieu Du Hoc at www.tailieuduhoc.org

Type

Examples Benjamin.

Notes of the verb, not objects. For example, in the following two sentences, we are comparing objects, carrots and potatoes, not the subject I.: I like carrots more than I do potatoes. I like carrots more than do I like potatoes. Now, in this sentence, we are comparing subjects, I and my friend Carl: I like carrots more than does my friend Carl.

5. intro comparative

Bigger than an apatosaur is the blue whale. More important than your personal statement is your GPA. No less impressive than the invention of the laser was the development of the wheel.

Inversion is obligatory. Used with be-verbs. This form is more common on the GMAT and GRE than it is on the TOEFL. Notice that we can only use this form of inversion when the verb is a be-verb since in every case, the comparative is the complement of the be-verb. Remember that less than is also a comparative.

6. as

Megumi is from Japan, as is Sato. So-eun wants to leave early today, as does Oi. If thrown into the water, camels

Inversion is obligatory. Used with all verbs. We can only use inversion if we are using as for comparisons. as is one of the trickiest words in

For more material and information, please visit Tai Lieu Du Hoc at www.tailieuduhoc.org

Type

Examples can swim, as can cats.

Notes English; it can have many different meanings.

7. so that

So happy was I that I bought flowers for everybody in class. So quickly did she leave that we did not even realize was gone. So rarely does a comet appear visible to the naked eye that when one does, it is considered a major event.

Question form is obligatory. Used with all verbs. This is not so common on the TOEFL, but is fairly common on the GMAT and GRE. The so that clause must before the verb in for this type of inversion.

8. had, should, were for if-clauses

Had I remembered Tomomis birthday, she wouldnt be mad at me now. Should you need a hand, I will be more than happy to help you. Were I you, I think I would study more for your exam tomorrow.

Inversion is obligatory. Used with all verbs. This is somewhat common on the TOEFL and more common on the GMAT and GRE. This type of inversion is kind of special. Notice that we can only use this type of inversion when we are using an if-clause. In other words, if is omitted: even though the word if does not appear in the clause, we still have the meaning of an ifclause. For more information, see had, should, were.

9. there is, there are, there exists, there comes, etc.

There is a good restaurant nearby. There comes a time in every persons life when she realizes

Inversion is obligatory. Usually used only with these verbs. This form of inversion is common on the TOEFL, GMAT, and GRE,

For more material and information, please visit Tai Lieu Du Hoc at www.tailieuduhoc.org

Type

Examples that she is responsible for her own happiness, not other people. Scientists hypothesize that there exists a certain type of particle that can travel faster than the speed of light.

Notes as well as in spoken and written English. Most people remember there is and there are. BUT we must also remember that there are other verbs that we can use instead of is and are. The most common ones are exist, come, and go.

10. here is, here are, here comes, here come

Here is some good food for you to try. Here are the books that I dont need anymore. Here comes the bus!

Inversion is obligatory. Usually used only with these verbs. You will probably not see this on the grammar section of the TOEFL or on the GMAT or GRE. It could, however, appear on the Listening Comprehension Section of the TOEFL. We use this form mostly in spoken English.

11. intro -ing

Burning out of control was the forest located in the foothills of the Sierra Nevada mountains. Coming in last in the race was Joe Elephant Legs Blow. Not helping the situation was little Susie, who was throwing newspaper on the spreading fire.

Inversion is obligatory. Used only with be-verbs. This form is not common on the TOEFL, but might show up on the GMAT or GRE. Notice the intro ing phrase is the complement of the be-verb.

12. emphasis

Boy am I hungry. Is it ever hot in here! Do you know how to cook!

Inversion is optional. Used with all verbs. You will probably not see this on the grammar section of the TOEFL

For more material and information, please visit Tai Lieu Du Hoc at www.tailieuduhoc.org

Type

Examples

Notes or on the GMAT or GRE. It could, however, appear on the Listening Comprehension Section of the TOEFL. We use this form mostly in spoken English.

13. the bigger, the better

The closer an object is to another object, the greater is the gravity between the two objects.

Question form is optional. Used with all verbs.

14. questions

Is this the last example? Do you enjoy reading these lists? Are we finished yet?

Inversion is obligatory. Used with all verbs. You will probably not see this on the grammar section of the TOEFL (TOEFL doesnt test questions anymore) or on the GMAT or GRE. It would, however, appear on the Listening Comprehension Section of the TOEFL.

15. "story speech"

I think its time to go, said Susan. Its time for you, but not for me, replied Gary. Maybe we should collect our thoughts for a moment, commented Lany.

Inversion is optional. Used with verbs that report speech. You will probably not see this on the grammar section of the TOEFL or on the GMAT or GRE.

16. nor

No one has volunteered for the job, nor do we expect anyone to volunteer in the future.

Inversion is obligatory. Used with all verbs. You might see this on the adaptive

For more material and information, please visit Tai Lieu Du Hoc at www.tailieuduhoc.org

Type

Examples Hok-ming cannot speak Portuguese, nor can Jos speak Cantonese. The zoo regulations will not permit you to touch the animals, nor would most people advise you to do so.

Notes TOEFL if you are scoring high and it could appear on the GMAT or GRE. Remember that nor is considered a conjunction, but we use it between two sentences (not between any two elements like the other conjunctions).

17. "so do I"/ "neither do I."

So do I. So can Terry. Neither do most people I know.

Inversion is obligatory. Used with all verbs. You will probably not see this on the grammar section of the TOEFL or on the GMAT or GRE.

18. intro adjective

Beautiful beyond belief was my baby daughter. Happy about their acceptance into their dream schools were Lany and Tomo. Quick and painless will be your medical procedure.

Inversion is obligatory in most cases. Used with be-verbs. This one is fairly rare and probably would not appear on the TOEFL, but you might see it on the GMAT or GRE. Inversion is sometimes not used in poetic language.

6) Parallelism (Apples + Oranges) a. When the sentence compares two items. Ask yourself, can they be really compared? b. When the sentence compares two actions as well.

Usually, the problem is with hidden comparison where two things or actions are compared, but another two items or actions are intertwined and you lose the comparison relationship.

For more material and information, please visit Tai Lieu Du Hoc at www.tailieuduhoc.org

Example: Synthetic oils burn less efficiently than natural oils. The sentence is wrong because we are trying to compare well each oil burns and not the oils themselves. But do you see how the actual thing being compared is easily missed? In a series of two or more elements, what you do on #2 determines what you do on 3+. In other words, everything after #2 must match #2: I like to swim, to run, and to dance. I like to swim, run, and dance.

are okay. I like to swim, run, and to dance. I like to swim, to run, and dance. are NOT okay.

7) Quantity Words a. The words measuring quantity may be used incorrectly. For example, when comparing two items, it would be inappropriate to use among to compare them. Heres a chart: 2 items Between More Better Less if more than 2 among most best least

b. Items that cant be counted should not use quantity words. For example, you cant say fewer soup. Heres a chart: Ok words for non-countable Less Amount, quantity Much c. Countable words fewer number many

When two distinct words or phrases are joined by the correlatives either, or, neither, nor, not only, but also, the number( singular or plural) of the word or phrase nearest to the verb determines the number of the verb. 1) Example: Either his parents or he is bringing it (notice is is singular) This can be a confusing sentence because parents is plural, but we pay attention to he which is the noun he tells us that we need to keep is singular.

For more material and information, please visit Tai Lieu Du Hoc at www.tailieuduhoc.org

2) Example: Either he or his parents are bringing it. Notice parents is plural and since this is the noun that is closest to the verb, we use are which is plural. 8) Idioms Definition: Incorrect usage of idiomatic expressions. There are no rules. Really need good English familiarity. Overall rule: If its not one of the previous 7, then its very likely an idiom expression test question. There are three types of idioms that you'll see on the GMAT: 1. word pairs that go together 2. prepositions and the verbs that use them 3. standard expressions.

Idiomatic preposition usage Many idiomatic expressions tested on the exam involve prepositions. There's no overarching grammatical rule that tells you which prepositions go with which verbs. Again, the rules are determined by usage, so you'll have to "listen" to the expression and determine if the verb is followed by the correct preposition. Be on the lookout for commonly tested prepositions like "of," "at," "by," "in," "from," "to," and "for". If you have difficulty determining whether a usage is correct, try testing out the idiom in a simpler version of the sentence. Example: Although he was considered as a leading proponent for the controversial new initiative, the professor nevertheless sought refuge from the media uproar. The sentence above becomes: The professor was considered as a proponent. Does anything sound unusual? Could this sentence be worded differently? In fact, to be, not as, is the correct idiom: The professor was considered to be a proponent. Would rather present tense if referring to myself and past tense if someone other than

the subject is doing the action Example: Would rather I speak Example: Would rather you spoke present tense past tense

Major Idioms you should be familiar with.

For more material and information, please visit Tai Lieu Du Hoc at www.tailieuduhoc.org

a debate over a lot a responsibility to a result of a sequence of accompanied by affect to.. (SC-47,pg 707 of OG) agree with alternative to as a result of...(SC-263,pg 761 of OG) as an instance of as good as/or better than as great as as good as...or better than as much as..(SC-105,pg 720 of OG) assume ...to be of... attend to (someone) attribute X to Y/X is attributed to Y based on believe X to be Y benefit from better served by X than Y ..(SC-116,pg 722 of OG) between X and Y (Sc-115, pg 722 of OG) both X and Y call...to consider... centers on concerned with conform to contrary to... created with credited with ..(SC-111,pg 721 of OG) decline in defined as depends on whether depicted as different from/differ from distinguishes between X and Y (SC-15,pg 699 of OG)

more X than Y ...(SC-175,pg 737 of OG) more...than ever...(SC-98,pg 718 of OG) must have (done) neither...nor no less....than noted that ..(Sc-258,pg 759 of OG) not in a flash...but in a ... (SC-22,pg 701 of OG) not only...but also not so much...as not X ...but rather Y ..(SC-172,pg 736 of OG) one attributes X (an effect) to Y (a cause) (SC-21,pg 701 of OG) One X for every ZZ( some numeric number) Y's ...(SC89,pg 717 of OG) persuaded X to do Y (sc-173,pg 736 of OG) prohibits X from doing Y potential to range from X to Y ...(SC-58,pg 710 of OG) range of ...(SC-77,pg 714 of OG) reason.. that incorrectly seen as reason.. because

regards X as Y ... regard as regardless replacing with... research to (SC-112, pg 721 of OG) responsible for resulting in restitution...for ...(SC-130,pg 726 of OG) retroactive to same to X as to Y (sc-54,pg 709 of OG) seem...to...(seem is plural) seem to indicate ...(SC-95,pg 718 of OG) so X as to be Y so (adjective) that so X that Y ...(SC-177,pg 737 of OG) subscribe to such...as targeted at ( SC-195,pg 742 of OG) that X ...that Y ...(SC-250,pg 757 of OG)

For more material and information, please visit Tai Lieu Du Hoc at www.tailieuduhoc.org

distinguish from doubt that either...or enable to estimated ...to be ...(SC-14,pg 699 of OG) entrusted with expected X to be Y expected that X would be Y ... extent to ...(SC-57,pg 710 of OG) fascinated by forbid X to do Y identical with forcing ...to... for jobs..(SC-109,pg 721 of OG) for over...XXX years... from X to Y had better(do) in contrast to independent from indifferent towards interaction of ...(SC-237,pg 753 of OG) modeled after (no) more...than/(no) less...than

the same to X as to Y to .. used to (example to get used to or to become used to) to contrast X with Y to monitor ...(SC-239,pg 754 of OG) to mistake X for Y (sc-150,pg 730 of OG) to orbit...(SC-49,pg 708 of OG) to result in to sacrifice X for Y to survive (SC-181,pg 738 of OG) used to (do) used in the construction... used to construct... viewed marriage as ( sc-147,pg 730 of OG) widely anticipated that.... X as Y ..(SC-182,pg 738 of OG) X is attributed to Y (SC-21,pg 701 of OG) X forbids Y to do Z ...(SC-100,pg 719 of OG) X [is] expected to Y (SC-131, pg 726 of OG) X ordered that Y be Z'ed...(SC-218,pg 748) X ordered Y to do Z (sc-121,pg 724 of OG) X ordered Y to be Z'ed..(SC-218,pg 748 of OG) X prohibits Y from doing Z ...(SC-100,pg 719 of OG)

Examples of Idioms in Sentences


A access to The company has access to large capital reserves. act as The poison pill in the contract acts as a preventative measure against hostile takeovers. allows for The design of the robot arm allows for great flexibility. as....as Chocolate tastes as good as ice cream. associate with He associates beer with potato chips. attribute to The poor first quarter results are attributed to the restructuring. a responsibility to The CEO has a fiduciary responsibility to all shareholders. a result of The recent Nasdaq decline is a result of higher interest rates. a sequence of The Sumerian text was a sequence of incomprehensible symbols. agree with The Teamsters do not agree with the Republicans on many issues. among Used when discussing more than two items. He was the finest policeman among the hundreds of rookies. as good as/or better than The new software is as good as or better than anything on the market

For more material and information, please visit Tai Lieu Du Hoc at www.tailieuduhoc.org

as great as The new house looks as great as I had hoped. attend to (someone) The emergency room doctor attended to the injured victim. attribute X to Y/X We attribute the results to the new management. attributed to Y The extinction of the dinosaurs has been attributed to an asteroid collision. B based on The results are based on a comprehensive ten year study. begin to He will begin to study twelve hours before the test. believe X to be Y After seeing the flying saucer, I believe UFOs to be a real phenomenon. between Used when discussing two things (if there are more than two, then use among instead). He could not decide between Corn Flakes or Raisin Bran. C care about How much do business schools care about your score? centers on + noun The GMAT centers on the knowledge of basic math and writing/reading skills. choose to The number of students who choose to go to business school has increased in the last ten years. consistent with Your grades are not consistent with your abysmal GMAT scores. contend that He contends that the GMAT has a cultural bias. consider + noun How important do you consider the test? continue + to If you continue to study, you will succeed. contrast A with B If you contrast A with B, you can see the difference. convert to You may convert muscle to fat if you study too much. compare A to B (compare to stresses similarities). The music critic favorably compared him to Bob Dylan. compare A with B (compare with stresses differences). Broccoli is good for you compared with ice cream. count on + noun He counts on management support. concerned with They are concerned with investor relations more than actual profitability. conform to When you work at a new company, you should try to conform to its corporate culture. D decide to We decided to continue. decide on We decided on the new format. depend on The global economy depends on improving productivity. different from The CAT is very different from the paper and pencil GMAT. difficult to Many students find the CAT difficult to take. distinguish between X and Y Distinguish between domestic and international production. distinguish X from Y Juries must attempt to distinguish truth from falsehood. depends on whether Our place in the playoffs depends on whether we win tonight. E to be + essential to + noun Speed is essential to success in the Internet marketplace. except for He did well on the GMAT, except for the sentence correction questions. F

For more material and information, please visit Tai Lieu Du Hoc at www.tailieuduhoc.org

flee from The convict fled from the country. G grow from Dell Computer grew from a start- up to a Fortune 500 company in less than fifteen years. grow out of Needless to say, they quickly grew out of their first office. H help + noun + to Their direct business model helped them to grow rapidly. I indicate that Dell's recent stock trouble may indicate that their growth will not continue to be as rapid. invest in He is too risk-averse to invest in the stock market identical with His DNA is identical with his twin's. in contrast to The candidate claims to support tax cuts, in contrast to his prior statements. independent from The Federal Reserve Board is supposed to be independent from political considerations. indifferent towards Some countries are indifferent towards human rights. L leads to Rapid growth often leads to problems. like Usually only used for direct comparison: He walks like Joe walks. localized in Most Internet venture capital is localized in a few areas of the world. M mistake + noun + for I mistook you for an old friend. modeled after The judicial building is modeled after the Parthenon. more than ever Companies demand MBA graduates now more than ever. N native to There is a unique business culture native to the U.S. a native of It infects those who are not even a native of America. need to Living in New York City is an experience everyone needs to try. to be + necessary + to It is necessary to get a high GMAT score to get into Stanford. neither...nor Neither Tom nor Sam has the necessary skills to finish the job. not only...but also Stanford not only has the highest GMAT average, but also the highest GPA. P prohibit from + gerund You are prohibited from using a calculator on test day. potential to A graduate of a top business school has the potential to make over $100,000. R range from X to Y The GMAT scores at top business schools will range from 650 to 750. refer to If you have any more questions, you should refer to a grammar book. regard as Wharton's finance program is regarded as the finest in the world. require + noun + to You require a GMAT score to go to most U.S. business schools. rivalry between X and Y The rivalry between the Boston Red Sox and the New York Yankees is one of the most celebrated in professional sports. responsible for The manager is responsible for seven entry level employees. retroactive to The tax policy change is retroactive to last year.

For more material and information, please visit Tai Lieu Du Hoc at www.tailieuduhoc.org

S save for Save for William, no one else passed the exam. save from Many people use business school to save them from dull jobs. so that So should not be used as an adjective: GMAT preparation is so... boring. Use it with "that." This guide is designed so that you may raise your score. subscribe to Business school students should subscribe to the Wall Street Journal. T tie to The contract should be tied to concessions. transmit to The communications system will transmit to anyone within range. U used + infinitive Japan used to be the model industrial economy. to be + used to + gerund After five practice tests, he was used to the GMAT CAT format.

Word Pairs (Examples)


asas morethan lessthan greater.than not onlybut (also) sothat She was not only exhausted but (also) famished as well. The apartment was so expensive that no self-supporting student could afford it. (Just) as it is the duty of employees to contribute to the well(just) asso being of the company, so it is the duty of the company to contribute to the well-being of its employees. neithernor eitheror Neither a borrower nor a lender be. The workshop was more thrilling than anything I'd ever done. The movie was as long as it was boring.

Word Usage
Some pairs of words, like fewer and less, are often used incorrectly because they're treated as synonyms. In fact, there is a solid rule that determines which one you should use, and the exam will test your ability to decide which is the correct option. The exam tests four such word pairs with particular frequency, so memorize the rules pertaining to them if you don't know them already. 1. Fewer/Less Which of the two following sentences is correct?

For more material and information, please visit Tai Lieu Du Hoc at www.tailieuduhoc.org

1. The company fired no less than fifty employees. 2. The company fired no fewer than fifty employees. The second sentence is correct. Why? Because you use less when you're talking about things you can't count (less pollution, less violence) but fewer when you're talking about things you can count (fewer pollutants, fewer violent acts). 2. Number/Amount These words follow the same rule as less and fewer. Number is correct when you can count the thing being described (a number of cars, a number of people) and amount is correct when you cannot (amount of love, amount of pain). 3. Among/Between Use between when only two options are available (between the red car and the blue car) and among when more than two options are available (among the five answer choices, among the many books). 4. If/Whether Whether is correct when you're discussing two options (whether to get chocolate or strawberry ice cream) and if is correct for more than two options (if she should get ice cream, frozen yogurt, or a cookie). 5. each other/one another each other refers to two things (Tom and Linda helped each other up the hill) one another refers to 3 or more things (Each member of the family supported one another) 6. Another/the other Another nonspecific other (This is another fine mess youve got me into) Other- specific other (During lunch, I ate one dessert and Linda ate the other one.)

Use of Being Important! Although many of these absolute phrases could be written with the word being in them, more formal English (and ETS!) tends not to use being when being is optional. If you've studied GMAT Sentence Correction for a while, then you know that the word being raises a big red flag on the test! Here are some examples: The movie being over, we left the theater. This sentence could be rewritten like this: The movie over, we left the theater.

For more material and information, please visit Tai Lieu Du Hoc at www.tailieuduhoc.org

People who study for GMAT for a while quickly learn that being is usually wrong.

So I'm guessing you already know that being in an answer choice is wrong more often than it is right.

This is a good strategy to get you started, but to get over 700 on the GMAT, you really need to know some of the finer points of GMAT Sentence Correction that relate to the use of the word

General Grammar definitions So= therefore So that=in order to/in order that That=the fact that

Conjunctions that can join two independent clauses are: and, but, yet, for, or, nor Dont start sentences with Because

Words requiring how (example: know how) Know Teach Learn Show

Always choose active over passive voice Active example: Elaine purchased new software for the company Passive example: New software was purchased for the company by Elaine Another frequently tested grammatical error is unnecessary use of the passive voice. It's a good idea to become familiar with this type of error; it appears quite often in the answer choices of sentence correction questions. As a reminder, the passive voice is in use when the action of the sentence is performed on the subject. The active voice is in use when the subject itself performs the action. Example: Active: Allison (subject) went (action) to the store to buy a cake (object). Passive: The cake (subject) was bought (action) by Allison (object).

For more material and information, please visit Tai Lieu Du Hoc at www.tailieuduhoc.org

Preposition use To or Of needs subject + verb NOT object + verb Examples of subject: she, he, whoever, who, I Examples of object: her, him, whomever, its, it

Either side of a form to be (were, was) must have subjects not objects on either side to agree.

A verb that ends in ing is a gerund which is a verb acting like a noun. Try not to use gerunds if you can help it.

Adverbs, not adjectives modify verbs Key identifier: Adverbs end in ly Correct example: I sure wish I were rich Incorrect example: I surely wish I were rich

being
There are at least two different situations in which being is often the right answer. Here is the first example of when being is correct: When the grammar requires it. Yes, I'm trying to simplify things here, but the idea is this--many ideas can be expressed in more than one way. For example, I can say:

I'm afraid of being late. I'm afraid that I'll be late.

Each has its own emphasis, but the point is that these two structures exist.

Whether we can express ideas in one or more structures is really related to the word used; in other words, it is idiomatic.

But some idioms allow only one structure. For example:

For more material and information, please visit Tai Lieu Du Hoc at www.tailieuduhoc.org

In addition to being one of the first restaurants to combine Mediterranean and American tastes, Chez Panisse in Berkeley is also one of the Bay Area's most established restaurants.

The idiomatic structure in addition to does not have a counterpart that uses a subject and a verb, so our only option here is to use being, which is grammatically a noun, but is derived from a verb.

The second example of when being is correct is shown in this example:

There are many reasons to get an MBA, with increased career prospects being the most important for many MBA applicants.

Technically this part here:

with increased career prospects being the most important for many MBA applicants

is an absolute phrase, but I think it's also helpful just to memorize the pattern:

with + NOUN + being + NOUN COMPLEMENT

Like vs. Such As Question: What's the difference between like and such as? Example of the "mistake" that we make in everyday speech: Can you buy me some fruit like oranges or grapefruit? How the GMAT Official Guide would explain this mistake: Using like in this answer choice mistakenly suggests that the utterer of the request does in fact not want oranges or grapefruit, but rather some other kind of fruit that is similar to oranges or grapefruit. In normal English: like means similar to, and such as means for example. Take a look at these examples: Can you buy me some fruit like oranges or grapefruit?

For more material and information, please visit Tai Lieu Du Hoc at www.tailieuduhoc.org

In GMATLand, this sentence would mean that you do NOT want oranges or grapefruit; instead, you'd prefer some fruit similar to oranges and grapefruit. For example, you may want pomelo, lemons, or limes. Yes, I know this sounds a little crazy, but our goal is to understand what GMAT is looking for, not what is "correct" English. Can you buy me some fruit such as oranges or grapefruit?

Yes, this is what we're supposed to say in GMATLand -- oranges and grapefruit are examples of the type of fruit we want. I would like you to buy such fruit as oranges and grapefruit for me, if you don't mind.

This is simply a variation -- notice how such and as are separated. Separating the two elements tends to make this pattern a bit harder to see.

What does Question form is obligatory mean? This simply means that you MUST invert the subject and the verb in this construction. In other constructions, inversion is optional, but in these constructions, it is required. For example, you may say: She runs faster than do most of her classmates. (verb comes before the subject) or She runs faster than most of her classmates do. (subject comes before the verb) BUT Never have I heard such a thing! CANNOT become XX Never I have heard such a thing! XX

Like vs. Such As


Question: What's the difference between like and such as? Example of the "mistake" that we make in everyday speech: Can you buy me some fruit like oranges or grapefruit? How the GMAT Official Guide would explain this mistake: Using like in this answer choice mistakenly suggests that the utterer of the request does in fact not want oranges or grapefruit, but rather some other kind of fruit that is similar to oranges or grapefruit. In normal English: In GMATLand, like means similar to, and such as means for example. Take a look at these examples: Can you buy me some fruit like oranges or grapefruit?

For more material and information, please visit Tai Lieu Du Hoc at www.tailieuduhoc.org

In GMATLand, this sentence would mean that you do NOT want oranges or grapefruit; instead, you'd prefer some fruit similar to oranges and grapefruit. For example, you may want pomelo, lemons, or limes. Yes, I know this sounds a little crazy, but our goal is to understand what GMAT is looking for, not what is "correct" English. Can you buy me some fruit such as oranges or grapefruit?

Yes, this is what we're supposed to say in GMATLand -- oranges and grapefruit are examples of the type of fruit we want. I would like you to buy such fruit as oranges and grapefruit for me, if you don't mind.

This is simply a variation -- notice how such and as are separated. Separating the two elements tends to make this pattern a bit harder to see.

What's the Difference Between that and which?


After people study GMAT sentence correction for a while, they may ask about the difference between that and which. I have to say for the record, though, that it is not terribly important to know the difference between the two to get a high score on the test, but I know that some people are naturally curious about the difference (people here tend to be overachievers!). A Tip But first, a testmagic tip: GMAT almost always (I say almost always because I've seen two questions that did not follow this rule, but the rule was violated in all five answer choices) wants you to put a comma before which. In other words, if you see which without a comma before it, it's probably wrong. After this explanation, you should understand why, but for those of you who want only the most important information, this is what you need to know. An Example Both of these sentences are correct in GMAT land: Please go into the room and get me the big book, which is mine. Please go into the room and get me the big book that is mine.

Yes, in GMATland, these two sentences have two different meanings. Both of these sentences would be incorrect in GMAT land: X Please go into the room and get me the big book which is mine. X X Please go into the room and get me the big book, that is mine. X

Notice the commas--that's what makes all the difference. The Explanation Okay, we have in English this weird idea that we need to use different grammar in an adjective clause (a.k.a. relative clause) depending on whether the information in the adjective clause is necessary to specifically identify which noun we are referring to.

For more material and information, please visit Tai Lieu Du Hoc at www.tailieuduhoc.org

For example, imagine you have one sister, and you are telling a friend that your sister is coming to visit you. Since this person is your friend, we can presume that he knows that you have only one sister. You utter a sentence like this to your friend in GMATland: "My sister, who just graduated from college, is coming to see me."

In GMATland, since your friend (we presume) knows you well and knows that you have only one sister, this extra bit of information is considered unnecessary to identify which sister it is you are talking about. It is a sort of "by the way" information--"My sister is coming to see me, and oh, by the way, she just graduated from college." Now imagine you have two, three, or even more sisters. Let's imagine that one is a college professor, another is a webmaster, and this one who is coming to visit you just graduated from college. If you're talking to your friend, and you say only "my sister," and you do not mention her name, your friend might not know which sister you are talking about. So you add that extra bit of information--my sister who just graduated from college--to identify which sister it is you are referring to. In this situation, we have just correctly employed a very important grammar rule. Read on. So, if the person you're talking to, or the person who's reading what you've written, needs that extra bit of information to know which noun you're referring to, we say that that extra information is non-restrictive. This word doesn't really describe the function clearly, so many teachers say that this information is "extra." On the other hand, if you need that information to know which noun you are talking about, we say that the information is restrictive. Again, this word is not really a good choice for clarity, and many teachers use the term "necessary information" instead. Finally, just to make English a bit more difficult, we have a rule that says we should use a comma before or after "extra information clauses and phrases," but not with "necessary information clauses or phrases." The idea here is that the comma represents the slight pause in speech or change in intonation that a native speaker might use when making such an utterance. I should point out that both that and which are relative pronouns, i.e., they are grammatically the same, but their meanings are slightly different. Now, let's return to our original example sentences: Please go into the room and get me the big book, which is mine.

In this sentence, the clause which is mine is "extra" because the information "the big book" is enough to identify which book it is that you want. We can assume that there is only one big book in the room. Please go into the room and get me the big book that is mine.

For more material and information, please visit Tai Lieu Du Hoc at www.tailieuduhoc.org

In this sentence, the clause that is mine is "necessary" because the information "the big book" is NOT enough to identify which book it is that you want--it is probably the case that there are several big books in the room, so I need to add the information "that is mine" to identify which book it is that I want. More Examples A few more examples may help: I met with Bill Clinton, who is a lawyer.

The name Bill Clinton is enough to identify which person I'm talking about--who is a lawyer is therefore extra information. I met with the man who is a lawyer.

In this case, "the man" is not enough information to identify which person I'm talking about--who is a lawyer is therefore necessary information. The Sun, which is the only star in our solar system, is the source of heat for Earth.

Again, the name "the Sun" already clearly identifies the noun; therefore, the information in the adjective clause "which is the only star in our solar system" is extra. The star that is at the center of our solar system is called what?

In this case, since we don't have a name here, we don't know which star it is that we are referring to. Therefore, the information in the adjective clause "that is at the center our solar system" is necessary. A good rule to learn is that which requires a comma before it. If you're not sure about why the comma is necessary, please see this post.

However, GMAT is very tricky at times, and can of course create a question in which this rule does NOT apply. In fact, I've seen a question that used which without a comma, but ALL the answer choices violated the rule. I.e., none of the answer choices used that; they all used which without a comma.

Here's an example of what I mean:

Lucise in San Francisco is a place which anybody can visit. (A) which anybody can visit (B) which any person could visit (C) which no person could not visit (D) which, if they wanted to, any person could visit (E) which any person could visit if they so desired

For more material and information, please visit Tai Lieu Du Hoc at www.tailieuduhoc.org

I know some of you will think that this sentence is terrible, but the whole point of GMAT sentence correction is that we must choose the best answer, NOT the perfect answer.

For more material and information, please visit Tai Lieu Du Hoc at www.tailieuduhoc.org

Critical Reasoning
On the verbal part of the GMAT, you will encounter about 14 Critical Reasoning questions of various lengths (sometimes you will even need to scroll to read all the answer choices). In 99 per cent of cases, you will have a short passage with one question right under it. The argument you meet can be anything from a classical argument to an advertisement or a dialog. The questions will ask you to manipulate the argument to weaken/strengthen it, find the conclusion, assumption, explanation, do an inference, supplement a statement, or even tell how its parts are related to each other. On average, you will have 1:50 for each question, but it is recommended that you try to stay within 1:30 on CR (Critical Reasoning) questions since you will need to save some time for Reading Comprehension.

It is recommended that you read through Kaplan's Verbal workbook or the Section of CR in the Kaplan GMAT book with CD. Both of the books are good for building a solid background; in our sessions, we will develop further the techniques described in these books, yet will not rely on them for exercises or anything else.

First of all, Critical Reasoning is ability to reason clearly to evaluate and judge arguments. You are using this skill a lot during you everyday life while reading newspapers or watching movies. When you think that the movie is pushing the limit of the Reasonable or the news sounds less reasonable than the movie that was pushing the limit, you are using your CR skills to produce these conclusions. Besides the Verbal part on the GMAT, you will also need good argumentative skills to beat the essays since one of them is to construct an argument, and the other is to evaluate one. As a rule, GMAT CR questions will ask you to manipulate the argument to weaken/strengthen it, find the conclusion, assumption, explanation, do an inference or supplement a statement, etc. Whatever it is that you have to do, you will need 2 things to succeed: know the basic structure of arguments and clearly understand the argument.

In general, about 80% of GMAT arguments consist of evidence, usually 2 pieces, a conclusion the main point of an argument, and an assumption - the bridge between the evidence and conclusion. The majority of the arguments you encounter on the test will be 3 step arguments: Evidence1 + Evidence2 = Conclusion.

For more material and information, please visit Tai Lieu Du Hoc at www.tailieuduhoc.org

Example 1. Last week Mike was detained for shoplifting at a groceries store near his house, but he has been a Christian for 10 years, therefore, the police must have been wrong accusing him in stealing. We have here two pieces of evidence: Mike was accused of stealing and that his is a Christian. The conclusion is that the police are wrong. Therefore, our huge assumption here is that a Christian could not have stolen anything.

Example 2. There are a lot of mosquitoes outside today, please do not turn on the light in the room because a lot of them will fly in. There is no set scheme for structure in GMAT CR, but since the majority of the arguments are only a few sentences long, the conclusion usually comes in the first or the last sentence. However, some of the arguments you encounter will not have a conclusion at all or will have just an implied one. Let's go through the strategy to approach CR questions; we will cover it today in general and then will practice some of the crucial steps one by one and then in the third session will consolidate the approach.

Critical Reasoning Strategy

For more material and information, please visit Tai Lieu Du Hoc at www.tailieuduhoc.org

[We want to warn you immediately that this strategy is not the easiest way to do CR (the easiest would be read-and-answer), but it lets you get the most questions right spending less time per correct answer. The bottom line is that it won't be easy to follow this strategy but if you do, it will reward you] 1. Read the passage (Read it very attentively because in contrast to Reading Comprehension, there is very little text here and mostly everything is important; try to read only once. Reread only hard texts). Identify any evidence or premises and the conclusion. Note, sometimes the conclusion is the question itself. ETS damn theyre tricky. 2. Paraphrase the passage (this a very important step because when you do a paraphrase, you check whether you understood the passage and at the same time you extract the skeleton of the argument, making it easier to identify the conclusion and the assumption. Very often, the paraphrase of the passage will be pretty close to the conclusion. It is not surprising, since the conclusion is the main point and evidence just supports it.) Your paraphrase should be as close to the text and as simple as possible so that you would understand it easily and at the same time could fully trust it. Do not make it too general nor too detail oriented. When you do a paraphrase, do it in three steps: Evidence1, Evidence2, and Conclusion; put "therefore" word before you start your conclusion, this will help you to set it off. 3. Read the question again (now with more understanding of what is being asked; reading the question 2 times, will also help you to make sure you answer exactly what is stated and that you understand the question.) 4. Answer before reading the answer choices (Why do this? Two reasons: one, if you can think of the correct answer or at least the general direction that the answer choice needs to be, you will identify it among the wrong choices much faster, thus spend less time reading the answers, which usually take 30 seconds to cover. The second reason is that often test takers are seduced by the author's wording. One reads a few words that were used in the passage and the brain identifies this choice with the passage, thus making it seem more right that it needs to be. The more problems you practice with, the more chance is you will guess the right answer even before reading it. And there is nothing more pleasant than seeing YOUR answer choices among the listed. ) 5. Go through the answers, first time scan them for YOUR answer choice (usually you will guess correctly in 60-70% of cases), if you did not find it, reread them more attentively. 6. Draw a grid to eliminate the wrong answers easier. Use "+" for a sure answer, "-" for a definitely wrong answer choice, and "~" or "?" for an answer that may be right or

For more material and information, please visit Tai Lieu Du Hoc at www.tailieuduhoc.org

questionable. This will help to concentrate only on a few answer choices and will prevent you from reading same answers several times if you get confused or keep having troubles locating the right answer. A B C D E ? + +

7. Always remember to think how the answer choice relates EXACTLY to this situation; it may be out of scope by being too general. E.g. Advertisement: For sinus pain, three out of four hospitals give their patients Novex. So when you want the most effective painkiller for sinus pain, Novex is the one to choose.

W hich of the following, if true, most seriously undermines the advertisement's argument?

(A) -(B) -(C) Many drug manufacturers increase sales of their products to hospitals by selling these products to the hospitals at the lowest price the manufacturers can afford. (D) Unlike some competing brands of painkillers, Novex is available from pharmacies without a doctor's prescription. (E) --

In this next section, we will be mostly concerned with one of the most important parts of a GMAT argument, Assumption.

Many CR questions directly ask for an assumption or are based on them, such as weaken and strengthen questions. Also, assumption of an argument is one of the only parts that we can influence to destroy or solidify an argument; thus if we can disprove an assumption, we can negate the whole argument because the conclusion will not make sense. On the other hand, if we can strengthen our assumption, thus patch a possible hole, we will create a stronger argument

For more material and information, please visit Tai Lieu Du Hoc at www.tailieuduhoc.org

and our conclusion will be more credible. (Sometimes some of the evidence is doubted or is amended to add new meaning and change the argument, but generally, it is the assumption that is attacked to destroy an argument. In a case when evidence is completed with more info, we can still say that it is the assumption that the evidence we had was valid is being destroyed). Thus, to succeed in CR, it is crucial that you are able to extract an assumption fast; it will save you time on many questions. Practicing with assumptions does not take too long; 2-4 hours and your skills will be on top. Also, you will be a better speaker and a more critical writer if you are able to see assumptions of the writer who argues against you because as we have said, if you can kill an assumption, you will damage the argument. For example, if we play with Example 1 and add a piece of evidence that says that Christians sometimes steal, we will not be able to say with confidence that the police are wrong about Mike being a thief, since evidence proves that Christians do steal.

We will also cover one of the most important steps of the CR approach, paraphrase. Paraphrase allows you to check how well you understood the passage as well as to see the structure stripped. Paraphrase should not take you more than 5-10 seconds after you have read the passage. Sometimes it is useful to write down some complicated relationship or something that will help you to understand the passage better, such as a diagram, for example. Other than that, don't write down anything else, the passage that you read will be so short that you should be able to remember all the details without writing down anything. Today we were supposed to learn that the assumption of an argument is a bridge (link) between the evidence and the conclusion of an argument never stated in the text is the most vulnerable element of an argument is the only element in the argument that can be influenced should be easy for you to find by now

Paraphrase Should be brief and take 5-10 seconds Will state the main idea of the passage and be close to the conclusion Will help you to understand the passage better Will reveal the conclusion, evidence, and eventually, the assumption Start your conclusion with Therefore

Homework

For more material and information, please visit Tai Lieu Du Hoc at www.tailieuduhoc.org

Work on the section 10 and 11 of the GMAT+ materials available for download at www.angelfire.com/scifi2/gmatplus/gmat.html, paraphrasing all passages and writing down the assumptions. Also, please, underline the conclusions; some of the passages won't have a conclusion/assumption; in this case, leave out the conclusion and assumption, but still do a paraphrase. To check your answers, go to the appendix of the guide: 1) Section 10 can be found in Appendix B 2) Section 11 can be found in Appendix C

Appendix A Free Scaled down version of information youll find in Deltacourse. Deltacourse is 10 times more thorough and covers more subcategories of Quant question types not covered well in Princeton or Kaplan.

Arranging Objects

The number of ways of arranging n unlike objects in a line is n! (pronounced n factorial). n! = n (n 1) (n 2) 3 2 1

Example How many different ways can the letters P, Q, R, S be arranged? The answer is 4! = 24.

This is because there are four spaces to be filled: _, _, _, _ The first space can be filled by any one of the four letters. The second space can be filled by any of the remaining 3 letters. The third space can be filled by any of the 2 remaining letters and the final space must be filled by the one remaining letter. The total number of possible arrangements is therefore 4 3 2 1 = 4! The number of ways of arranging n objects, of which p of one type are alike, q of a second type are alike, r of a third type are alike, etc is: n!/(p! q! r!)

Example

For more material and information, please visit Tai Lieu Du Hoc at www.tailieuduhoc.org

In how many ways can the letters in the word: STATISTICS be arranged? There are 3 Ss, 2 Is and 3 Ts in this word, therefore, the number of ways of arranging the letters are: 10! = 50 400 3! 2! 3!

Rings and Roundabouts

The number of ways of arranging n unlike objects in a ring when clockwise and anticlockwise arrangements are different is (n 1)! When clockwise and anti-clockwise arrangements are the same, the number of ways is (n 1)!

Example

Ten people go to a party. How many different ways can they be seated? Anti-clockwise and clockwise arrangements are the same. Therefore, the total number of ways is (10-1)! = 181 440 Combinations The number of ways of selecting r objects from n unlike objects is: nCr = n!/r! (n r)!

Example

There are 10 balls in a bag numbered from 1 to 10. Three balls are selected at random. How many different ways are there of selecting the three balls? 10C3 = 10! = 10 9 8 = 120 3! (10 3)! 3 2 1

Permutations

A permutation is an ordered arrangement. The number of ordered arrangements of r objects taken from n unlike objects is: nPr = n!/(n r)!

For more material and information, please visit Tai Lieu Du Hoc at www.tailieuduhoc.org

Example

In the Match of the Days goal of the month competition, you had to pick the top 3 goals out of 10. Since the order is important, it is the permutation formula which we use. 10P3 = 10!/7! = 720

There are therefore 720 different ways of picking the top three goals.

Probability

The above facts can be used to help solve problems in probability.

Example

In the National Lottery, 6 numbers are chosen from 49. You win if the 6 balls you pick match the six balls selected by the machine. What is the probability of winning the National Lottery? The number of ways of choosing 6 numbers from 49 is 49C6 = 13 983 816 . Therefore the probability of winning the lottery is 1/13983816 = 0.000 000 071 5 (3sf), which is about a 1 in 14 million chance

The probability of an event occurring is the chance or likelihood of it occurring. The probability of an event A, written P(A), can be between zero and one, with P(A) = 1 indicating that the event will certainly happen and with P(A) = 0 indicating that event A will certainly not happen.

Probability = the number of successful outcomes of an experiment the number of possible outcomes So, for example, if a coin were tossed, the probability of obtaining a head = , since there are 2 possible outcomes (heads or tails) and 1 of these is the successful outcome.

Using Set Notation

Probability can be studied in conjunction with set theory, with Venn Diagrams being particularly useful in analysis.

For more material and information, please visit Tai Lieu Du Hoc at www.tailieuduhoc.org

The probability of a certain event occurring, for example, can be represented by P(A). The probability of a different event occurring can be written P(B). Clearly, therefore, for two events A and B, P(A) + P(B) - P(AnB) = P(AuB) P(AnB) represents the probability of A AND B occurring. P(AuB) represents the probability of A OR B occurring.

Mutual Exclusive Events

Events A and B are mutually exclusive if they have no events in common. In other words, if A occurs B cannot occur and vice-versa. On a Venn Diagram, this would mean that the circles representing events A and B would not overlap.

If, for example, we are asked to pick a card from a pack of 52, the probability that the card is red is . The probability that the card is a club is . However, if the card is red it can't be a club. These events are therefore mutually exclusive. If two events are mutually exclusive, P(AnB) = 0, so P(A) + P(B) = P(AuB)

Independent Events

Two events are independent if the first one does not influence the second. For example, if a bag contains 2 blue balls and 2 red balls and two balls are selected randomly, the events are: a) independent if the first ball is replaced after being selected b) not independent if the first ball is removed without being replaced. In this instance, there are only three balls remaining in the bag so the probabilities of selecting the various colours have changed. Two events are independent if (and only if): P(AnB) = P(A)P(B) This is known as the multiplication law.

Conditional Probability

For more material and information, please visit Tai Lieu Du Hoc at www.tailieuduhoc.org

Conditional probability is the probability of an event occurring, given that another event has occurred. For example, the probability of John doing mathematics at A-Level, given that he is doing physics may be quite high. P(A|B) means the probability of A occurring, given that B has occurred. For two events A and B, P(AnB) = P(A|B)P(B) and similarly P(AnB) = P(B|A)P(A). If two events are mutually exclusive, then P(A|B) = 0 .

Example

A six-sided die is thrown. What is the probability that the number thrown is prime, given that it is odd. The probability of obtaining an odd number is 3/6 = . Of these odd numbers, 2 of them are prime (3 and 5). P(prime | odd) = P(prime and odd) = 2/6 = 2/3 P(odd) 3/6

For more material and information, please visit Tai Lieu Du Hoc at www.tailieuduhoc.org

Appendix B

Critical Reasoning: Section 10 Please download section 10 from the gmatclub.coms CR section Read through passages, but do not answer the questions, dont even try. What you need to concentrate first is paraphrasing and pinpointing assumptions to check yourself, below you will find paraphrases and assumptions for each question in chapter 10

Question # 3

Paraphrase Old study eating chocolate increases chances of getting heart disease; new study chocolate does not increase heart disease chances; therefore, people will buy more chocolate

Assumption People were not buying chocolate because they were afraid of heart disease

As climate of North America got hot and dry large mammals died while small ones survived

This assumption is possible only after brining in an implied conclusion change in climate killed the large animals. Large mammals were more sensitive to heat than small ones

Higher profits will give higher bonuses, therefore, general economic recession year will bring lower bonuses than year of profits

The Industry will not bring profits in the year of general economic recession

Theres less suitable area for gray wolves, however, bringing gray wolves back to the places where they have been hunted out is immoral

People will keep hunting

Developed a safer equipment for Swedish market, brought it to US but doesnt advertise this safety improvement

Safety improvement should be advertize because it will help him win over its competitors

For more material and information, please visit Tai Lieu Du Hoc at www.tailieuduhoc.org

Few residents use public buses because there are enough automobiles, therefore, public bus system should not be subsidized

Killing subsidy for the public bus system will not hurt the suburbs community

14

A survey was conducted, according to is employees with high ratings ware satisfied with the companys system, therefore, the companys best performing employees like the system

Employees with high ratings are the companys best performing employees

16

1970s fall in the average annual income of college graduates as compared to high school graduates; 1980s the average annual income of college graduates increases while the number of college graduates did not decrease

N/A

19

Industrialists are accused of intervention in war to make profit, however, federal expenses for intervention were larger than profits, therefore, the accusation is wrongly motivated

Federal money (expenses and profits) and industrialists money come from the same source

For more material and information, please visit Tai Lieu Du Hoc at www.tailieuduhoc.org

Appendix C

Critical Reasoning: Section 11 Please download section 11 from the gmatclub.coms CR section Read through passages, but do not answer the questions, dont even try. What you need to concentrate first is paraphrasing and pinpointing assumptions to check yourself, below you will find paraphrases and assumptions for each question in chapter 11

Question # 1.

Paraphrase School board decided to reduce its staff by laying off the least effective teachers first

Assumption It is possible to determine the level of teachers effectiveness

2.

Applied scientific research is emphasized because it leads to technological advancement; basic research should be paid attention to because its a base for applied research

Basic research does not lead to technological advancement

3.

Decision to invest in electronic system has cost advantage over nonelectronic system, therefore, it will give advantage over competitors

Competitors are not using the electronic system yet and wont match it

5.

One museum sold 30K tickets, in a year 2 more museums opened and together the 3 of them sold 80K tickets, therefore, museums were worth the cost

The first old museum did not sell more tickets next year, as compared to previous year, and it is due to the two new museums that the number of tickets sold increased

8.

Company needs to cut costs, so offer early retirement first, and then fire others to have the overall reduction of costs to 50 percent

Reducing costs by firing will not hurt companys productivity: 50 percent of managers left will be able to do the double work load

For more material and information, please visit Tai Lieu Du Hoc at www.tailieuduhoc.org

9.

Need to reduce airport congestion, so send passengers by rapid trains to several cities while realizing that it is the major airport that is congested

Passengers will be willing to switch from air travel to trains

12.

Received 2000 letters, most of them support him, therefore, most people in the country support him

People who wrote the letters represent the opinion of majority

13.

Government wanted to regulate what can be shown in tobacco and alcohol advertisement, as a result, these ads became more inventive and humorous

Implied conclusion: tobacco and alcohol advertisement should be banned. 1. Use of tobacco and alcohol can be influenced by advertisements 2. The funny and inventive commercials are more persuasive than the old ones.

15.

Lead contamination dropped: federal regulation went into effect, but mainly because there was a drop in the use of leaded gasoline

N/A no conclusion

17.

Decrease in traditional child disease, and at the same time increase in rare infection among children whereas few adults are affected

Children are more sensitive to rare infections than adults

18.

Plants come from less developed nations without compensation, but coal, oil and ores are extracted for payment

The value and benefit from these plants are the same as from coal, oil, etc. They are comparable

19.

Donates an exhibit which demands a storage space, a routine conservation,

The number of visitors to see the exhibit will not override the financial

For more material and information, please visit Tai Lieu Du Hoc at www.tailieuduhoc.org

therefore, it adds to museums expenses and does not help it financially

costs of keeping the exhibit

20.

Oil prices are remaining low, therefore, natural gas prices will also be low

Oil and natural gas prices are interconnected

For more material and information, please visit Tai Lieu Du Hoc at www.tailieuduhoc.org

Appendix D
General Strategies for Reading Comprehension, by Stephen Bolton 1. Try to read the whole text of the passage once, if possible. Many people think you should just skim the passage or read the first lines of every paragraph, and not to read the passage. We believe this is an error: if you misunderstand the main idea of the passage, you will certainly get at least some of the questions wrong. Give the passage one good read, taking no more than 3 minutes to read all of the text. Do not read the passage more than once that wastes too much time. If you have not understood it completely, try to answer the questions anyway. Note: this point of reading the whole passage is important for test-takers whose first language is not English, provided that they can read the passage in 3 minutes or less.

2. Make brief notes on the text on your scrap paper. As we will see below in greater detail, you should write down a couple of words on A) the Main Idea or Primary Purpose, B) Organization/Structure of the passage, and C) the Tone or Attitude of the author (if applicable). You just need a few words for each of these areas, and altogether it should not take longer than 30 seconds to write down.

3. Remember that the tone or attitude of the passage is usually respectful and moderate, never going to extremes of praise nor criticism. ETS obtains its Reading Comprehension passages from real articles about real academics and professionals. So the tone of the articles, even when there is criticism in the passage toward an academic or her work, is always balanced and moderate. In the same vein, articles that deal with minorities or ethnic groups are almost always positive and sympathetic.

4. Look out for structural words that tell you the important ideas or transitions in a passage.

Continue the Idea Words Similarly Moreover

For more material and information, please visit Tai Lieu Du Hoc at www.tailieuduhoc.org

Additionally In the same way Likewise

Conclusion Words Thus Therefore Hence So In summary In conclusion

Contradiction or Contrast Words Neverthless Nonetheless However But Although Though Even though Notwithstanding Yet Despite In spite of On the one handon the other hand While Unlike

5.

Go back to the text of the passage for the answers. Many test-takers fail to return to the text of the passage to look for the correct answers. They rely solely on their memories and understanding of the passage after having read or skimmed it. Wrong. ETS is counting on that. Go back to the text to look for information to answer the questions. Nine times out of ten, the answer lies within the passage.

For more material and information, please visit Tai Lieu Du Hoc at www.tailieuduhoc.org

Of the 6 most important types of questions for Reading Comprehension, we will first look at Main Idea/Primary Purpose Questions, and the strategies we can use to answer them.

Main Idea/Primary Purpose Questions Many people believe there is no difference between the main or central idea of the passage and the primary purpose of the author of the passage. This is simply not true. Let's take a look at the subtle but important difference between them: Main Idea The question might look something like this:

"Which of the following best states the central idea of the passage?" "Which of the following most accurately states the main idea of the passage?" "Which of the following is the principal topic of the passage?" "The main topic of the passage is...."

Primary Purpose The question might look like this:

"The primary purpose of this passage is to..." "The primary purpose of the passage as a whole is to..." "The primary focus of this passage is on which of the following?" "The main concern of the passage is to..." "In the passage, the author is primarily interested in...." "The passage is chiefly concerned with..."

Strategy: Main Idea: Look in the first and last paragraphs for the main idea. Any conclusion words like therefore, thus, so, hence, etc. that you see are most likely introducing the main idea. The correct answer will say the same thing as it says in the text, but using different words. The Main Idea is not always stated explicitly in the passage in fact, more likely than not, it is not stated explicitly. Therefore, in order to answer this type of question when it is more implicit: 1. Re-read the first line of every passage, and the last line of the first and last paragraphs. This should give you the general structure or outline of the argument, with which you can answer the Main Idea question.

For more material and information, please visit Tai Lieu Du Hoc at www.tailieuduhoc.org

2.

After determining the general structure or content of the argument, eliminate answer choices that are too broad or too specific, i.e. answer choices that go beyond the content of the passage, or that deal with content only discussed in one paragraph of the passage.

3.

Make brief notes a couple of words- regarding the Main Idea on the text on your scrap paper while you read.

Primary Purpose: What is the author trying to do? What is his intention? If he is evaluating a theory, then the answer could be something like "Discuss an interpretation". Note that the correct answer would deal with "an interpretation", because the author is only dealing with one theory. If the Primary Purpose is to criticize 2 new books, then his intention or his primary purpose might be to "Critique new studies". Again, as in Main Idea questions, re-read the first line of every passage, and the last line of the first and last paragraphs. This should give you the general structure or outline of the argument, with which you can answer the Primary Purpose question. Note: A good main idea or primary purpose does not go beyond the scope of the passage, nor does it limit itself to discussing only one part of the passage.

What is the primary purpose of this passage? A) B) C) D) E) discuss the importance of the television program Star Trek for the international space program discuss important theoretical work concerned with faster-than-light space travel. explore a dispute among theoretical physicists regarding the uses of space flight describe the possible uses of space-warping material explain how a space-warping bubble would work in the real world

Explanation This is a Primary Purpose question, so we have to determine what the author is trying to do or say in this passage. So, let's read the first and last lines of the passage in order to get an idea of the primary purpose. The first line says "Great news for Star Trek fans: warp drives that can propel starships around the Galaxy faster than the speed of light may be possible after all--with a little help from Dr Who." The last line is a quote by a physicist that says "Of course, there are still some basic questions--like how does one go about constructing this Tardis space-time--but it puts the concept of space warps back on the agenda." From both these sentences, we get the idea of space travel, faster than light travel and space warps maybe this is a discussion of faster than light space travel. Does that match what you have already read? Yes, basically this is a discussion of the theoretical state of play in the area of faster-than-light space travel. Do any of

For more material and information, please visit Tai Lieu Du Hoc at www.tailieuduhoc.org

the 5 answer choices match that? Yes B, even if the wording is somewhat different from how we are wording it, the idea is almost exactly the same. B is the answer. Another way of getting to the answer is through elimination of obviously incorrect answer choices. We can eliminate A because the author mentions the popular science fiction program Star Trek merely to introduce the idea of faster-than-light travel, and nothing more. C is a stronger possibility because the second paragraph of the passage does discuss some disagreement among physicists about the possibility of creating a warp-drive, but in the same paragraph the theoretical dilemma seems resolved. Moreover, since the author only discusses this in one paragraph, it cannot be the primary purpose of the entire passage. We can eliminate D because the author does not go into detail discussing the uses of space-warping material. And we can discard E because the author does not really go into how the space-warping bubble would work in the real world.

For more material and information, please visit Tai Lieu Du Hoc at www.tailieuduhoc.org

Title Questions by Stephen Bolton, 20th August, 1999 Title questions are very similar to Main Idea questions, though are less common. Though some of the example paassage we use in this tutorial and in the Practice Section are from the New Scientist, and therefore have titles, the passages in the real GMAT will not have titles. The question might look like this:

"Which of the following titles best summarizes the passage as a whole?"

Strategy: Treat this as a Main Idea question. A good title sums up the central idea of a passage. Therefore, in order to answer this type of question:

1.

Look in the first and last paragraphs for the main idea. Any conclusion words like therefore, thus, so, hence, etc. that you see are most likely introducing the Main Idea/Title. The correct answer will say the same thing as it says in the text, but using different words.

2.

Re-read the first line of every passage, and the last line of the first and last paragraphs. This should give you the general structure or outline of the argument, with which you can answer the Title question.

3.

Make brief notes a couple of words- regarding the Title on the text on your scrap paper while you read.

4.

After determining the general structure or content of the argument, eliminate answer choices that are too broad or too specific, i.e. answer choices that go beyond the content of the passage, or that deal with content only discussed in one paragraph of the passage.

What would be an appropriate title for this passage? A) B) C) D) Constructing The Tardis How To Make Space-Warping Material Bubbles In Space-Time Faster-Than-Light Travel: A Possibility?

For more material and information, please visit Tai Lieu Du Hoc at www.tailieuduhoc.org

E)

Debate On The Uses of Space Travel

Explanation This passage actually already has a title, "Warp Factor One". But we have to look for another title possibility, one that would be most like the Main Idea of the passage. We look at the first and last paragraphs, and since the Main Idea is that researchers now feel that faster-than-light travel maybe more than mere fantasy, we can find the correct answer choice. Does any answer choice corrspond to this idea? Yes- answer D, which is the correct answer. We can also find the correct answer through elimination. There is nowhere in the passage where it discusses building Dr. Who's Tardis (pity!), so we can eliminate A. Nor does it tell us how to make space-warping material. Eliminate B. While bubbles in space-time are discussed at some length in one of the paragraphs, we cannot say this is the main concern of the passage, and thus should eliminate C. And nowhere are the uses of space travel discussed, so discard E.

Specific Detail or Target questions are probably the most common types of questions, and the easiest to answer. The question might look like this:

"According to the passage,...." "The passage states that ...."

Strategy The Specific Detail or Target that we are looking for could be a Line Number, or a Name or Date. Go to the Line Number or Name or Date, and then read several lines above and below it. Find the answer choice that basically says the same thing as in the passage, though usually with different words or word order.

According to the passage, Pfenning and Ford A) B) C) D) E) demonstrated conclusively the impossibility of faster-than-light travel explored the possibility of bubbles that warp space supported the work of Alcubierre work at of the Institute for Theoretical Physics at the Catholic University of Leuven suggested that a warp drive was not physically possible

For more material and information, please visit Tai Lieu Du Hoc at www.tailieuduhoc.org

Explanation This is a Specific Detail/Target question, and therefore we look for the Name, Line Number, or Date that will help us. In this case, the detail consists of the names Pfenning and Ford. We scan the text, starting from the top of the passage, looking for the names Pfenning and Ford. We find them in only place, at the beginning of the second paragraph. We read a couple of lines above the names, and keep reading until a few lines after the names. It says "But in 1997 Michael Pfenning and Larry Ford at Tufts University in Medford, Massachusetts, apparently killed this ingenious idea by showing that it needed far more than the entire energy content of the Universe to work (This Week, 26 July 1997, p 6)". The line after that says the research of another physicist then resurrected the possibility of FTL travel, negating the implications of the research of Pfenning and Ford. Now we can answer the question. Do any of the answer choices match the information given around the target area? Yes- E. Let's also eliminate. If we re-read what the passage says about Pfenning and Ford, we can eliminate B, C, and D. None of them are supported by the information in the passage, so let's eliminate all of them without wasting too much time and with a minimum of fuss. A is tougher to eliminate. From the sentence that mention Pfenning and Ford, it seems their work does rule out the possibility of a space-warp drive. But if we read the next line, it says another researcher said it was indeed possible. So the Pfenning and Ford could not have "conclusively" demonstrated the impossibity of the FTL drive.

This is probably the most difficult type of Reading Comprehension problem. The question might look like this: "It can be inferred that the author makes which of the following assumptions?" "Which is an assumption underlying the last sentence of the passage?" "Which of the following, if true, would most strengthen the hypothesis mentioned in lines 17-19?" "With which of the following statements regarding chaos theory would the author be most likely to agree?"

Strategy: 1. First, treat this type of problem as a Specific Target question. Look for a target in the question, find it in the text, and then look above and below it. Often you do not have to infer very much, the answer remains within the text. 2. If the answer must be inferred and is not stated explicitly within the text, then choose the answer choice that can be inferred or assumed from the information given. Again, you

For more material and information, please visit Tai Lieu Du Hoc at www.tailieuduhoc.org

should not have to infer very much only one or two logical steps removed from the information in the passage. 3. Make sure that the answer choice you decide on does not violate or contradict the Main Idea of the passage - if it does, the answer choice is probably wrong.

It can be inferred that a house with the propeties of the bubble mentioned in the passage A) B) C) D) E) would be larger on the inside than on the outside could move faster than the speed of light might be very energy efficient could move through time would eventually fold in on itself and be destroyed

Explanation First, let's try to deal with this question as a Specific Target problem. Is there a target in the question? Yes the bubble. The bubble is first mentioned at the end of the second paragraph, and then discussed at length throughout the third paragraph. Remember, we have to look above and below that target area (as well as read the target area again), so quickly go through the second, third, and first part of the fourth paragraph. When you are finished, look at the answer choices. Can any of them be inferred from the information given in the target area? Well, we could eliminate C, D, and E for simply not being supported by the information given in the passage. B maybe, but a house moving through time seems pretty silly. But in the fourth paragraph the author talks about the Tardis, "which looked like a police box but had a spacious interior". Big on the inside, small on the outside. Is that like our house? Yes- answer A. As well, we can choose A because it does not go against or contradict the Main Idea in this case, which if it had, would have made it necessary to eliminate. So choose A.

The question might look like this: "The author's attitude towards Morgan's theory could best be described as one of ..." Strategy: Look for descriptive words, adjectives or adverbs, that could tell you the author's attitude. For example, the words unfortunately or flaw suggest a negative connotation, while strength or valuable emphasize the positive. Make brief notes a couple of words- regarding the Tone of the text on your scrap paper while you read. Additionally, keep in mind that the author's attitude

For more material and information, please visit Tai Lieu Du Hoc at www.tailieuduhoc.org

toward a theory, book, or ethnic group will almost always be respectful, even when somewhat critical.

The author's attitude towards Miguel Alcibierre's theory could best be described as one of A) B) C) D) E) Admiration mild skepticism unbridled scorn Dismay complete objectivity

Explanation Since this is a Tone/Attitude question, we must look in the passage for descriptive words that tell us what the author thinks of Alcibierre and his theory. In the second paragraph the author call's Alcibierre's theory "this ingenious idea". This is positive, and the only positive answer choice is A. A is the correct answer. As well, if we could not find the tone so easily, we could also eliminate C and D at the very least, for being too extreme.

The question might look like this: "Which of the following best describes the organization of the passage?" "Which of the following best describes the organization of the first paragraph of the passage?" "One function of the third paragraph is to...." Strategy: Re-read the first line of every passage, and the last line of the first and last paragraphs. This should give you the general structure or outline of the argument, with which you can answer the question. Remember to make brief notes about the structure of the text on your scrap paper. If you are looking for the organization of one paragraph, read the first and second sentence of the paragraph. That will give you a rough idea of what is the structure or organization of the paragraph. Which of the following best describes the organization of the second paragraph of the passage? A) B) C) D) Two investigations that support Alcubierre's theory are introduced Possible objections to the uses of the warp drive are present, and then refuted An objection to the practicality of the theory is raised, and then another work is cited to shore up the applicability of the original theory A work of theoretical physics that supports Alcubierre's theory is raised, and then another

For more material and information, please visit Tai Lieu Du Hoc at www.tailieuduhoc.org

that refutes it is presented E) Alcubierre's theory is analyzed by a panel of several eminent physicists

Explanation Read the first sentence of the paragraph: "But in 1997 Michael Pfenning and Larry Ford at Tufts University in Medford, Massachusetts, apparently killed this ingenious idea by showing that it needed far more than the entire energy content of the Universe to work (This Week, 26 July 1997, p 6)". Then read the second sentence: "Now Chris Van Den Broeck of the Institute for Theoretical Physics at the Catholic University of Leuven, Belgium, has resurrected Alcubierre's proposal". So if we out those two sentences together, and in different words, first the usefulness of Alcubierre's theory is questioned by two researchers, then the theory is validated by yet another researcher. Which of the answer choices is closest to this? C. None of the other answer choices follow the organizational pattern of the paragraph they reverse it, or are completely dissimilar. C is the only possible answer.

1. 2. 3.

Read the whole text of the passage once. Make brief notes about the text on your scrap paper. Remember that the tone or attitude of the passage is usually respectful and moderate, never going to extremes of praise nor criticism.

4.

Look out for structural words that tell you the important ideas or transitions in a passage.

5.

Go back to the text of the passage for the answers to specific questions.

Das könnte Ihnen auch gefallen